If m and n are integers, is m+m^2-n an even number?

This topic has expert replies
User avatar
Elite Legendary Member
Posts: 3991
Joined: Fri Jul 24, 2015 2:28 am
Location: Las Vegas, USA
Thanked: 19 times
Followed by:37 members

Timer

00:00

Your Answer

A

B

C

D

E

Global Stats

[Math Revolution GMAT math practice question]

If m and n are integers, is m+m^2-n an even number?

1) m is an even number
2) n is an even number

User avatar
GMAT Instructor
Posts: 1449
Joined: Sat Oct 09, 2010 2:16 pm
Thanked: 59 times
Followed by:33 members

Timer

00:00

Your Answer

A

B

C

D

E

Global Stats

Max@Math Revolution wrote:[Math Revolution GMAT math practice question]

If m and n are integers, is m+m^2-n an even number?

1) m is an even number
2) n is an even number
$$m,n\,\,\,{\rm{ints}}\,\,\left( * \right)$$
$$m + {m^2} - n\,\,\mathop = \limits^? \,\,{\text{even}}\,\,\,\,\,\,\mathop \Leftrightarrow \limits^{\left( {**} \right)} \,\,\,\,\boxed{\,\,n\,\,\mathop = \limits^? \,\,{\text{even}}\,\,}$$
$$\left( ** \right)\,\,m + {m^2} = m\left( {m + 1} \right)\,\,\mathop = \limits^{\left( * \right)} \,\,{\rm{even}}$$
$$\left( 1 \right)\,\,m\,\,{\rm{even}}\,\,\,\left\{ \matrix{
\,{\rm{Take}}\,\,\left( {m,n} \right){\rm{ = }}\left( {{\rm{0}}{\rm{,0}}} \right)\,\,\,\, \Rightarrow \,\,\,\left\langle {{\rm{YES}}} \right\rangle \hfill \cr
\,{\rm{Take}}\,\,\left( {m,n} \right){\rm{ = }}\left( {{\rm{0}}{\rm{,1}}} \right)\,\,\,\, \Rightarrow \,\,\,\left\langle {{\rm{NO}}} \right\rangle \hfill \cr} \right.$$
$$\left( 2 \right)\,\,n\,\,{\rm{even}}\,\,\, \Rightarrow \,\,\,\left\langle {{\rm{YES}}} \right\rangle $$

The correct answer is therefore (B).

This solution follows the notations and rationale taught in the GMATH method.

Regards,
Fabio.
Fabio Skilnik :: GMATH method creator ( Math for the GMAT)
English-speakers :: https://www.gmath.net
Portuguese-speakers :: https://www.gmath.com.br

GMAT/MBA Expert

User avatar
GMAT Instructor
Posts: 16207
Joined: Mon Dec 08, 2008 6:26 pm
Location: Vancouver, BC
Thanked: 5254 times
Followed by:1268 members
GMAT Score:770

by Brent@GMATPrepNow » Wed Oct 31, 2018 5:20 am

Timer

00:00

Your Answer

A

B

C

D

E

Global Stats

Max@Math Revolution wrote:[Math Revolution GMAT math practice question]
If m and n are integers, is m + m² - n an even number?

1) m is an even number
2) n is an even number
Given: m and n are integers

Target question: Is m + m² - n an even number?
This is a good candidate for rephrasing the target question
Aside: Here's a video with tips on rephrasing the target question: https://www.gmatprepnow.com/module/gmat- ... cy?id=1100
We can take the expression, m + m² - n and rewrite it as m(1 + m) - n
Now notice that m and (1+m) are CONSECUTIVE integers, which means one value, either m or (m+1), is EVEN, and the other value is ODD
So, we can be certain that the product m(1 + m) is EVEN
So, our question becomes Is (some even number) - n even?
In order for this value to be even, we need n to be even
So, we can REPHRASE the target question.....
REPHRASED target question: Is n EVEN?

Statement 1: m is an even number
Since we're given no information about n, we cannot answer the REPHRASED target question
Statement 1 is NOT SUFFICIENT

Statement 2: n is an even number
Perfect! The answer to the REPHRASED target question is YES, n IS even
Since we can answer the target question with certainty, statement 2 is SUFFICIENT

Answer: B

Cheers,
Brent
Brent Hanneson - Creator of GMATPrepNow.com
Image

User avatar
Elite Legendary Member
Posts: 3991
Joined: Fri Jul 24, 2015 2:28 am
Location: Las Vegas, USA
Thanked: 19 times
Followed by:37 members

by Max@Math Revolution » Fri Nov 02, 2018 12:19 am

Timer

00:00

Your Answer

A

B

C

D

E

Global Stats

=>
Forget conventional ways of solving math questions. For DS problems, the VA (Variable Approach) method is the quickest and easiest way to find the answer without actually solving the problem. Remember that equal numbers of variables and independent equations ensure a solution.

The first step of the VA (Variable Approach) method is to modify the original condition and the question. We then recheck the question.

m+m^2-n = m(m+1) - n. Now, m(m+1) is an even number since m(m+1) is the product of two consecutive integers. Thus, the parity of m+m^2-n depends on the parity of n only.
Thus, condition 2) is sufficient.

Therefore, B is the answer.
Answer: B